LSAT and Law School Admissions Forum

Get expert LSAT preparation and law school admissions advice from PowerScore Test Preparation.

User avatar
 Dave Killoran
PowerScore Staff
  • PowerScore Staff
  • Posts: 5852
  • Joined: Mar 25, 2011
|
#94178
Complete Question Explanation
(The complete setup for this game can be found here: viewtopic.php?f=385&p=94172)

The correct answer choice is (C)

If K supervises two employees, and G supervises two employees, the employees must be in the 1-1-3 distribution, with G and K as the president and manager, not necessarily in that order. The other three employees are each technicians. Note also that answer choices (D) and (E) both reflect the 1-2-2 distribution.
 Paul Marsh
PowerScore Staff
  • PowerScore Staff
  • Posts: 290
  • Joined: Oct 15, 2019
|
#78031
This is a Local Must be True question. It gives us the new rule that K supervises exactly two employees. So combined with our old Rule about G needing to supervise two employees, we now have both K and G each supervising exactly two employees. This means that K can't be a technician. And for the same reason that G can't be a manager in the 1-2-2 distribution, K ALSO now can't be a manager in the 1-2-2 distribution (again, this is because the other manager would then have nobody to supervise, which violates our rules). So if K and G both can't be managers in the 1-2-2 distribution (and we know neither one can ever be a technician), that would mean that K and G would both have to be president. But there's only room for one president, so that means that the 1-2-2 distribution isn't possible in this Local question. So we know we're in the 1-1-3 distribution.

In this 1-1-3 distribution (since again, K and G still can't be technicians), we know that the president is either K or G and the manager is either K or G. That means the other three employees (F, H, and L) are all technicians. Since this is a Must be True question, I'm interested in the variables that I can definitively place. Here, that's F, H, and L. So I scan my answer choices for one of those variables, and sure enough, (C) jumps right out as my right answer. (Since L is a technician, L must be supervised).
User avatar
 azzedineissa
  • Posts: 1
  • Joined: Jun 12, 2021
|
#87851
Thanks for this answer and the technique of thinking already about what variables will be locked into place by this new rule. Admittedly this question is tricky but brings to light the importance of keeping in mind the effect that variable placement and other conditions have on the distribution of spots along groups in open grouping games. This can speed up an answer and also help avoid a mistake. So thanks again.

Get the most out of your LSAT Prep Plus subscription.

Analyze and track your performance with our Testing and Analytics Package.